5
$\begingroup$

Hello All


Consider a matrix with elements:

$A_{i,j}=x_i$ for $i=j$
$A_{i,j}=1$ for $i\neq j$

Is there a closed form expression for the elements of $A^{-1}$?

Will be glad to know of any reference.

Thanks

HC

$\endgroup$
3
  • $\begingroup$ This question mathoverflow.net/questions/67924/… was on the eigenvalues of $D+J$ rather than the inverse, but it may be helpful. $\endgroup$ Jun 18, 2011 at 20:08
  • $\begingroup$ have a look here: people.kyb.tuebingen.mpg.de/suvrit/work/dimath/bm --- that link covers a slightly more general case than covered by your question. $\endgroup$
    – Suvrit
    Jun 19, 2011 at 3:53
  • 2
    $\begingroup$ Aùazing, the number of MO questions whose answer is an application of the Sherman-Morrison formula. $\endgroup$ Jun 19, 2011 at 7:25

1 Answer 1

14
$\begingroup$

You can use the Sherman-Morrison formula.

In the notation of the Wikipedia article, let $u=v=(1,\ldots,1)'$ and $A$ (not the same as your $A$) be the diagonal matrix with $(x_{1}-1, \ldots, x_{n}-1)$ on the diagonal.

Then, if I haven't made a mistake, the entry of the inverse matrix you're looking for is

$\frac{1}{x_{i}-1} - \frac{ \frac{1}{(x_{i}-1)^{2}} }{ 1 + \sum_{k} \frac{1}{x_{k}-1}}$ if $i=j$, and $ \frac{ -\frac{1}{ (x_{i}-1)(x_{j}-1) }}{ 1 + \sum_{k} \frac{1}{x_{k}-1}}$ otherwise.

$\endgroup$
3
  • $\begingroup$ Thanks Douglas for the reference. As I was working on the problem from your hint, Ian has already posted the solution! Thanks Ian. That's exactly I was looking for. Appreciate your help. $\endgroup$
    – user15871
    Jun 18, 2011 at 20:33
  • 1
    $\begingroup$ You might also want to rearrange the formulas I give to make it clearer that they work fine if one of the $x_{i}$ equals one. (Obviously the matrix is not invertible if two or more of the $x_{i}$ equal one.) $\endgroup$
    – Ian Martin
    Jun 18, 2011 at 20:34
  • $\begingroup$ Yes, I noticed that. Interestingly, I used round-about method to reach the solution yesterday, but that was not elegant and it was bothering me (solved for n=2, 3 and extrapolated from there by induction). But your approach is simple and elegant, and right use of Sherman-Morrison formula. Glad I posted the problem on this site. $\endgroup$
    – user15871
    Jun 18, 2011 at 20:40

Your Answer

By clicking “Post Your Answer”, you agree to our terms of service and acknowledge you have read our privacy policy.

Not the answer you're looking for? Browse other questions tagged or ask your own question.